• Anúncio Global
    Respostas
    Exibições
    Última mensagem

exercicio resolvido

exercicio resolvido

Mensagempor adauto martins » Qui Mai 06, 2021 12:18

(ITA-1951)achar o volume de uma piramide regular de base quadratica cuja diagonal mede 4 m.e cuja aresta lateral mede 1,5m.
adauto martins
Colaborador Voluntário
Colaborador Voluntário
 
Mensagens: 1171
Registrado em: Sex Set 05, 2014 19:37
Formação Escolar: EJA
Área/Curso: matematica
Andamento: cursando

Re: exercicio resolvido

Mensagempor adauto martins » Qui Mai 06, 2021 13:06

soluçao

aqui temos uma piramide regular(base de poligonos regulares,ou seja mesma medida dos lados),em nosso caso um quadrado.
mas a piramide nao pode ser reta(caso do ponto da base da altura estar no centro do poligono),pois aresta lateral e menor que 2m,ponto de encontro das diagonais.vamos a soluçao

o volume de uma piramide é dado por

{V}_{p}=(1/3).{A}_{b}.h

{A}_{b}={l}^{2}...

onde {A}_{b} area da base,
h,l altura e base respectivamente...

vamos calcular l

as diagonais se interceptam ao meio,logo teremos
triagulos-retangulos isosceles de lados iguais a 2m...tomemos um triangulo e usando pitagoras teremos...

{l}^{2}={2}^{2}+2^2=8\Rightarrow l=2.\sqrt[]{2}

{A}_{b}={l}^{2}={(2.\sqrt[]{2})}^{2}=8...

agora vamos calcular a altura h...como dito,a piramide é nao reta,logo a base da altura nao esta no centro do quadrado,pois a aresta lateral de medida 1,5 é menor que 2...mas continua na reta que liga os pontos medios de lados opostos(mostre isso,aqui é usar o centro de gravidade da piramide...)
a piramide tera duas faces laterais iguais e duas outras faces diferentes...as duas faces iguais sao triangulos retangulos,mostra-se usando o criterio de semelhança LAL,sao as faces que contem a aresta lateral de 1,5...
tomemos uma dessas faces,teremos entao um tringulo-retangulo de medidas 1.5,2,x...x a determinar...usando pitagoas teremos

x=\sqrt[]{(2.\sqrt[]{2})^2-(3/2)^2}=\sqrt[]{8-(9/4)}

x é o segmento dessa face,que une o vertice ao lado e perpendicular a esse...
tomemos o triangulo constituido por x,h,e o ponto da base da altura que sera a metade do ponto medio do lado do quadrado,que mede
\sqrt[]{2}...

logo,usando pitagoras teremos





x=\sqrt[]{23/4}=\sqrt[]{23}/2...

{x}^{2}={h}^{2}+{\sqrt[]{2}}^{2}\Rightarrow h=\sqrt[]{(\sqrt[]{(23}/2))^2-2}

h=\sqrt[]{(23/4)-2)}=\sqrt[]{15}/2...

{V}_{p}=(1/3).{A}_{b}.h=(1/3).8.\sqrt[]{15}/2=(4/3)\sqrt[]{15}...
adauto martins
Colaborador Voluntário
Colaborador Voluntário
 
Mensagens: 1171
Registrado em: Sex Set 05, 2014 19:37
Formação Escolar: EJA
Área/Curso: matematica
Andamento: cursando

Re: exercicio resolvido

Mensagempor adauto martins » Qui Mai 06, 2021 18:25

correçao

x=\sqrt[]{23}/2 nao é o segmento que une o vertice ao lado do quadrado,perpendicular a esse,e sim a outra aresta lateral...entao,vamos calcular esse segmento(chamaremos de y...).usando o criterio de semelhança LAL,teremos

y/(\sqrt[]{23}/2)=1.5/2\sqrt[]{2}=(3/2)/2\sqrt[]{2}

\Rightarrow y=(3/8).\sqrt[]{23}...

agora tomemos o triangulo com y=(3/8).\sqrt[]{23},l=\sqrt[]{2},h,determinaremos h.usando pitagoras teremos

h=\sqrt[]{{((3/8).\sqrt[]{23}})^{2}-{(\sqrt[]{2}})^{2}}

h=\sqrt[]{79/64}=\sqrt[]{79}/8

{V}_{p}=(1/3).8.(\sqrt[]{79}/8)=\sqrt[]{79}/3...
adauto martins
Colaborador Voluntário
Colaborador Voluntário
 
Mensagens: 1171
Registrado em: Sex Set 05, 2014 19:37
Formação Escolar: EJA
Área/Curso: matematica
Andamento: cursando


Voltar para Geometria Espacial

 



  • Tópicos relacionados
    Respostas
    Exibições
    Última mensagem

Quem está online

Usuários navegando neste fórum: Nenhum usuário registrado e 3 visitantes

 



Assunto: Taxa de variação
Autor: felipe_ad - Ter Jun 29, 2010 19:44

Como resolvo uma questao desse tipo:

Uma usina de britagem produz pó de pedra, que ao ser depositado no solo, forma uma pilha cônica onde a altura é aproximadamente igual a 4/3 do raio da base.
(a) Determinar a razão de variação do volume em relação ao raio da base.
(b) Se o raio da base varia a uma taxa de 20 cm/s, qual a razão de variação do volume quando o raio mede 2 m?

A letra (a) consegui resolver e cheguei no resultado correto de \frac{4\pi{r}^{2}}{3}
Porem, nao consegui chegar a um resultado correto na letra (b). A resposta certa é 1,066\pi

Alguem me ajuda? Agradeço desde já.


Assunto: Taxa de variação
Autor: Elcioschin - Qua Jun 30, 2010 20:47

V = (1/3)*pi*r²*h ----> h = 4r/3

V = (1/3)*pi*r²*(4r/3) ----> V = (4*pi/9)*r³

Derivando:

dV/dr = (4*pi/9)*(3r²) -----> dV/dr = 4pi*r²/3

Para dr = 20 cm/s = 0,2 m/s e R = 2 m ----> dV/0,2 = (4*pi*2²)/3 ----> dV = (3,2/3)*pi ----> dV ~= 1,066*pi m³/s


Assunto: Taxa de variação
Autor: Guill - Ter Fev 21, 2012 21:17

Temos que o volume é dado por:

V = \frac{4\pi}{3}r^2


Temos, portanto, o volume em função do raio. Podemos diferenciar implicitamente ambos os lados da equação em função do tempo, para encontrar as derivadas em função do tempo:

\frac{dV}{dt} = \frac{8\pi.r}{3}.\frac{dr}{dt}


Sabendo que a taxa de variação do raio é 0,2 m/s e que queremos ataxa de variação do volume quando o raio for 2 m:

\frac{dV}{dt} = \frac{8\pi.2}{3}.\frac{2}{10}

\frac{dV}{dt} = \frac{16\pi}{15}